LSAT and Law School Admissions Forum

Get expert LSAT preparation and law school admissions advice from PowerScore Test Preparation.

 Administrator
PowerScore Staff
  • PowerScore Staff
  • Posts: 8917
  • Joined: Feb 02, 2011
|
#25637
Complete Question Explanation
(The complete setup for this game can be found here: lsat/viewtopic.php?t=7800)

The correct answer choice is (A)

If one of the two visits to M is third, the other visit cannot be second or fourth in compliance with
the third and fifth rules:
june07_game_3_#16_diagram_1.png
Upon further inspection, you should realize that the range of possible solutions is wide open. This
is because the M > G > M sequence can be satisfied in weeks 1-2-3, but also in weeks 3-4-5 and
3-5-6. Rather than create three local diagrams, focus on the question at hand: when asked which
variables could be fourth and fifth, respectively, check to see which variables cannot be fourth
and fifth. In other words, proceed by the process of elimination:

From the first rule, we know that J cannot be visited fourth, eliminating answer choice (B). And,
since no destination is visited in two consecutive weeks, M cannot be visited fourth. This easily
eliminates answer choice (C).

Answer choice (D) cannot be true, because visiting J requires visiting G (not T) immediately
before J. While T could be fourth and J could be fifth in two different solutions, together they
create a TJ Block, which is impossible to satisfy.

Answer choice (E) is also impossible, because it creates an MTM Block in positions 3-4-5, which
violates the third rule.

With four of the answer choices eliminated, answer choice (A) is the only viable contender. For
the sake of efficiency, you need not verify that G and T could be fourth and fifth, respectively:
this answer choice must be correct, as none of the other answers provided an accurate list of
variables that can occupy these two spaces together. Here, this solution is tested for pedagogical
purposes only:
june07_game_3_#16_diagram_2.png
G and T are fourth and fifth, respectively, then J must be visited second to allow for the
creation of a GJ Block. Also, M must be visited sixth in compliance with the M > G > M
sequence established by the third rule. Thus, we arrive at a viable solution to the game:
june07_game_3_#16_diagram_3.png
You do not have the required permissions to view the files attached to this post.
 saranash1
  • Posts: 168
  • Joined: May 21, 2013
|
#9654
Section 1
16. For each answer choice there are two answers & they are the answers for the destinations in week 4 and five. For each answer choice is the first destination the destination for week 4 and the second destination listed the destination for in week 5? Or does it not matter the order?
I guessed B because g could go 4th and j could go 5th.
 Ron Gore
PowerScore Staff
  • PowerScore Staff
  • Posts: 220
  • Joined: May 15, 2013
|
#9690
Thanks for your question, saranash1.

By including the word "respectively" in the question stem, i.e., "...destinations in week 4 and week 5, respectively?", the test makers clarify that the left-hand column of destinations reflects week 4, and the right hand column reflects week 5. So, the order does matter. If you think you might get confused while answering the question, it may help you to write a "4" over the left-hand column and a "5" over the right-hand column. I imagine it was very frustrating to try and answer this question without having first fixed the columns in accordance with the question stem.

To more broadly help you with the question, remember that it is typically simpler to attack a Could Be True question by eliminating the incorrect answer choices. This is more efficient because you can easily waste a great deal of time thinking through the many various things that "could be true."

However, because the logical opposite of "could be true" is "cannot be true," we know that the four wrong answers are each express violations of the rules. It is typically easier to identify violations of the rules than it is to find one of the many situations the rules permit to occur.

In this case, (A) is the only answer that does not violate the rules. Answer (B) is incorrect because it violates the rule that J cannot be in week 4.

Hope that helps.

Ron
 saranash1
  • Posts: 168
  • Joined: May 21, 2013
|
#9719
so respectively means 4 first five second?
 Steve Stein
PowerScore Staff
  • PowerScore Staff
  • Posts: 1153
  • Joined: Apr 11, 2011
|
#9728
Hi saranash,

Thanks for your response--that is exactly right.

~Steve
 saranash1
  • Posts: 168
  • Joined: May 21, 2013
|
#9759
ok thanks

Get the most out of your LSAT Prep Plus subscription.

Analyze and track your performance with our Testing and Analytics Package.